Nbme 5 spoiler alert!!!!!!!!

This forum made possible through the generous support of SDN members, donors, and sponsors. Thank you.

RedSoxSuck

Full Member
15+ Year Member
Joined
Jan 30, 2009
Messages
755
Reaction score
6
2. A 22 year old woman has had progressive weakness over past 8 weeks. CBC: HB 8, leukocyte count 15k, segmented neutrophils 65%, lymphocytes 25%, monocytes 10%, Rouleaux present and platelet count is 60k.
which of the additional findings?

a. ANCA
b. Heterophile AB
c. Increase LAP activity
d. light chain protein...seems to be the right answer
e. 9;22t

I think its D because of rouleaux and lymphocyte. Obviously i am an idiot and i picked C.

3. A study is done to determine the relationship between use of oral contraceptives and cervical cancer. Study subjects include 50,000 women who are using oral contraceptives and 50,000 women who have had a tubal ligation. After 2 years of follow-up, the rate of in situ cervical cancer is 18 per 10,000 in the oral contraceptive group and 3 per 10,000 (p<0.05) in the tubal ligation group. Which of the following is the estimated relative risk of cervical cancer among users of OCP as compared with women who have had tubal ligation?

A) 3+18=0.17
B) 18+3=6
C) 18-3=15
D) 18+3=21
E) Indeterminable form the data given

4. A 16-year-old boy participates in a study of exercise and muscle hypertrophy. After a pertaining biopsy specimen of the quadriceps muscle is obtained, he begins a weight-lifting program to increase the mass and strength of the quadriceps muscle. After 3 months of training, a second muscle biopsy specimen is taken for histologic assessment of skeletal muscle characteristics. Compared with the pretraining sample, the post-training sample is most likely to show which of the following findings?



Number of Mitochondria....... Amount of Actin per Myocyte ......Total number of Myocytes



a) increase..... increase..... increase

b) increase.... increase....... no-change

c) increase.... no-change.... increase

d) increase no-change no-change

e) no-change increase increase

f) no-change increase no-change

g) no-change no-change increase

h) no-change no-change no-change

5. A homeless 45-year-old man comes to the emergency department because of pain in his right foot for the past week and difficulty walking for the past 2 days. A painful subcutaneous swelling is present in the right groin. His sock is blood-encrusted. A 5-cm (2-in) wound surrounded by swelling is present on the ball of the foot. A purulent exudate containing some blood is expressed on palpation of the wound. Which of the following is the most likely cause of the groin swelling?

A ) Cellular proliferation in an inguinal lymph node
B ) Localized edema from a protein-deficient diet
C ) Obstruction of lymphatic drainage in vessels near the knee
D ) Obstruction of venous return from the right leg
E ) Recirculation of neutrophils from the wound site

6. A patient with blood group 0 undergoing an operation for repair of an aortic aneurysm is mistakenly transfused with a group A unit of packed red blood cells. He immediately goes into shock A blood sample is markedly hemolyzed Which of the following is the primary mechanism for Ihe hemolysis?

A) Action by properdin lo cleave C3
B) Action of C5 9 complex of complement
C) Activation o( factor XII (Hageman factor)
D) Produciion of C5a to stimulate phagocytosis
E) Release of histamine by the action of C5a

i chose e because of anaphylactic shock or may be reasoning is faulty but it appears to be wrong. Correct answer seems to be b.

7. During bacteriophage T7 infection of a single E.coli cell, 1000 bacteriophage particles are released. One of these particles contains a fragment of the E.coli chromosome rather than the T7 chromosome. Will this phage be able to inject its DNA into E.Coli cell, and if so, how many T7 particles will be produced by this cell?

Able to inject DNA #of T7 particles produced
A Yes 1000
B Yes 100
C Yes 1
D Yes 0
E No 0

8. A 61-year-old woman has had progressively severe pain in the right ear over the past 24 hours. She has had type 1 diabetes mellitus for many years and has mild renal failure, retinopathy, and neuropathy. She has no history of drainage from the ear, upper respiratory tract infection, fever, or chills. She is afebrile. Hearing is normal bilaterally. The external appearance of the ear is normal, but touching or moving the pinna produces severe pain. The external auditory canal is inflamed. The tympanic membrane is clear, and no fluid is seen. Which of the following is the most likely diagnosis?

A ) Acoustic schwannoma
B ) Acute otitis media
C ) Labyrinthitis
D ) Malignant otitis externa
E ) Polychondritis

I put C but it seems like its wrong and right answer seems to be D

9. A 55-year-old Native American woman who lives alone is brought to the emergency department after a minor fall in her home, which her mother witnessed. She tells her physician that her husband, who died 6 weeks ago, comes to visit her every night and gives her guidance. She also states that he complains of being lonely and is waiting for her to join him. She states that her appetite and energy are normal and denies suicidal ideation. Physical examination shows a bruised arm and leg. Neurologic examination and CT scan of the head show no abnormalities. Which of the following is the most appropriate next step for the physician?

A ) Ask her relatives if speaking to deceased loved ones is common to their culture
B ) Ask her relatives if there is a family history of major depressive disorder with psychotic symptoms
C ) Ask her relatives if there is a family history of schizophrenia
D ) Order an EEG to determine if the patient is sleep-deprived
E ) Order an MRI of the head

I chose C. Not sure if its correct.

10. A 7-year-old child with mental ******ation is seen because of severe behavior problems in school and self-directed aggressive behavior The patient is diagnosed with a complete deficiency of hypoxanthme-guanme phosphoribosyltransferase. This defect most likely results in the accumulation of which of the following metabolites in serum?
A) Adenine
B) Cytosme
C) Guanine
D) Urea
E) Uric acid

I chose C because HGPRT breaks down guanine so i figured it will be accumulated. But it seems like answer is E. So E is more correct than C because you are most likely to see the symptoms presented due to accumulation of uric acid?

11. A previously healthy 21-year-old man is brought to the emergency department by friends 30 minutes after the sudden onset of right-sided chest pain and difficulty breathing while playing basketball. He appears in acute distress. He is 183cm (6ft) tall and weighs 65kg (143lb); BMI is 19kg/m2. His temperature is 37C (98.6F), pulse is 110/min, respirations are 32/min, and blood pressure is 120/70mmHg. Pulmonary examination shows hyperressonant sounds over the right lateral portion of the chest. Breath sounds are decreased. A mild pleural rub is heard. A chest x-ray shows a sharp line running parallel to the chest wall adjacent to a radiolucent area without lung markings. Which of the following is the most appropriate next step in management?

A) Spiral CT scan of the chest
B) MRI of the chest
C) Bronchoscopy
D) Intubation and mechanical ventilation
E) Needle aspiration

I chose C because i thought its tension pneumothorax. i am just curious because the answer might be E.

12. A 65-year-old man comes to the physician 2 weeks after noticing a nodule on his neck. He is an avid golfer, but he does not always use sunscreen while playing. Examination of the neck shows a 10-mm, pearly pink, raised nodule. An excisional biopsy specimen shows basal cell carcinoma. Which of the following is the most appropriate treatment?

a) Tretinoin therapy

b) Cryotherapy

c) Ionizing radiation therapy

d) UV irradiation

e) Wide excision and lymph node dissection

I think its b but can't it also be d? and why one answer is correct over the other.

13. 55 yr old woman comes to the physician c/o increased facial hair growth over past 8 yrs,menopause occured at age 45
testosteone - 80 ng/dl (N 10-90)
dihydroepiandrostenedione - 320mcg/dl(60-350)
estradiol 20pg/ml
progesterone - decreased
TSH 0.8mcU/ml

CAUSE OF INCREASED HAIR GROWTH?

A.decreased 5 alpha reductase activity
b decreased adrenal androgen production
c.decreased peripheral androstenedione conversion
d.decreased inhibin production
e.increased ovarian androgen production
f.incresed testosterone to estrogen ratio

I chose e but answer could be f. I did it online so i am not sure.


Thank you!!!!!
 
2. A 22 year old woman has had progressive weakness over past 8 weeks. CBC: HB 8, leukocyte count 15k, segmented neutrophils 65%, lymphocytes 25%, monocytes 10%, Rouleaux present and platelet count is 60k.
which of the additional findings?

e. 9;22t
Monocytes would not be increased in MM spectrum disorders. This could be myelomonocytic leukaemia. (going by the anaemia and thrombocytopenia) Possibly t(9;22) then?

3. A study is done to determine the relationship between use of oral contraceptives and cervical cancer.
Am I missing something here? Seems straightforward enough. your a/(a+b) and c/(c+d) values are already given to you, just divide. 18/3 = 6.

4. A 16-year-old boy participates in a study of exercise and muscle hypertrophy.
Number of Mitochondria....... Amount of Actin per Myocyte ......Total number of Myocytes
b) increase.... increase....... no-change

5. A homeless 45-year-old man comes to the emergency department because of pain in his right foot for the past week and difficulty walking for the past 2 days.
A ) Cellular proliferation in an inguinal lymph node

6. A patient with blood group 0 undergoing an operation for repair of an aortic aneurysm is mistakenly transfused with a group A unit of packed red blood cells.
B) Action of C5 9 complex of complement
The histamine itself wouldn't cause hemolysis, so I'm inclined to go with this option myself.

7. During bacteriophage T7 infection of a single E.coli cell, 1000 bacteriophage particles are released.
Able to inject DNA #of T7 particles produced
D Yes 0

Usually in generalised transduction the virus has no DNA of its own, so while the infection transfers genetic material, it is entirely bacterial, and no further viral propagation will occur.

8. A 61-year-old woman has had progressively severe pain in the right ear over the past 24 hours. She has had type 1 diabetes mellitus

D ) Malignant otitis externa
Ps. aeruginosa causes malignant OE especially in diabetic patients.

9. A 55-year-old Native American woman who lives alone is brought to the emergency department after a minor fall in her home, which her mother witnessed. She tells her physician that her husband, who died 6 weeks ago, comes to visit her every night and gives her guidance. She also states that he complains of being lonely and is waiting for her to join him. She states that her appetite and energy are normal and denies suicidal ideation. Physical examination shows a bruised arm and leg. Neurologic examination and CT scan of the head show no abnormalities. Which of the following is the most appropriate next step for the physician?

A ) Ask her relatives if speaking to deceased loved ones is common to their culture
B ) Ask her relatives if there is a family history of major depressive disorder with psychotic symptoms
Although it seems like a normal grief reaction to me, some of these behavioural science questions can be tricky. It's definitely not schizophrenia, this is not an auditory hallucination, and there are no symptoms suggestive of schizophrenia.

10. A 7-year-old child with mental ******ation is seen because of severe behavior problems in school and self-directed aggressive behavior
E) Uric acid

HGPRT also breaks down hypoxanthine to inosine. The key point is that guanine does not really build up, if there is excess it is metabolised by XO into uric acid.

11. A previously healthy 21-year-old man is brought to the emergency department by friends 30 minutes after the sudden onset of right-sided chest pain and difficulty breathing while playing basketball.
E) Needle aspiration

I chose C because i thought its tension pneumothorax. i am just curious because the answer might be E.
Now is not the time for curiosity, doctor. That comes later. Tension pneumothorax = decompress.


13. 55 yr old woman comes to the physician c/o increased facial hair growth over past 8 yrs,menopause occured at age 45

CAUSE OF INCREASED HAIR GROWTH?
f.incresed testosterone to estrogen ratio
I'm quite confused but I think due to the drop in SHBG the effective free test should be higher so the T/E ratio goes up?
 
Last edited:
Top